Difference between revisions of "2019 AIME II Problems/Problem 13"

(Created blank page)
 
Line 1: Line 1:
 +
==Problem==
 +
Regular octagon <math>A_1A_2A_3A_4A_5A_6A_7A_8</math> is inscribed in a circle of area <math>1.</math> Point <math>P</math> lies inside the circle so that the region bounded by <math>\overline{PA_1},\overline{PA_2},</math> and the minor arc <math>\widehat{A_1A_2}</math> of the circle has area <math>\tfrac{1}{7},</math> while the region bounded by <math>\overline{PA_3},\overline{PA_4},</math> and the minor arc <math>\widehat{A_3A_4}</math> of the circle has area <math>\tfrac{1}{9}.</math> There is a positive integer <math>n</math> such that the area of the region bounded by <math>\overline{PA_6},\overline{PA_7},</math> and the minor arc <math>\widehat{A_6A_7}</math> of the circle is equal to <math>\tfrac{1}{8}-\tfrac{\sqrt2}{n}.</math> Find <math>n.</math>
  
 +
==Solution==
 +
 +
==See Also==
 +
{{AIME box|year=2019|n=II|num-b=12|num-a=14}}
 +
{{MAA Notice}}

Revision as of 18:13, 22 March 2019

Problem

Regular octagon $A_1A_2A_3A_4A_5A_6A_7A_8$ is inscribed in a circle of area $1.$ Point $P$ lies inside the circle so that the region bounded by $\overline{PA_1},\overline{PA_2},$ and the minor arc $\widehat{A_1A_2}$ of the circle has area $\tfrac{1}{7},$ while the region bounded by $\overline{PA_3},\overline{PA_4},$ and the minor arc $\widehat{A_3A_4}$ of the circle has area $\tfrac{1}{9}.$ There is a positive integer $n$ such that the area of the region bounded by $\overline{PA_6},\overline{PA_7},$ and the minor arc $\widehat{A_6A_7}$ of the circle is equal to $\tfrac{1}{8}-\tfrac{\sqrt2}{n}.$ Find $n.$

Solution

See Also

2019 AIME II (ProblemsAnswer KeyResources)
Preceded by
Problem 12
Followed by
Problem 14
1 2 3 4 5 6 7 8 9 10 11 12 13 14 15
All AIME Problems and Solutions

The problems on this page are copyrighted by the Mathematical Association of America's American Mathematics Competitions. AMC logo.png